0
$\begingroup$

Mike wants to invest in $X_1$ if and only if he invests into $X_2$ or $X_3$ or both.

Please help i can't get my head around this

Thanks

$\endgroup$
1
  • $\begingroup$ Something like $X_1\geq \frac12\left(X_2+X_3 \right), \ \ X_1,X_2,X_3\in\{0,1\}$. But additional the objective function has to be known. $\endgroup$ Commented Nov 14, 2018 at 13:15

1 Answer 1

0
$\begingroup$

The constraint $$ X_1 \geq \frac12 (X_2 + X_3) $$ would ensure that $X_1$ is one if either is one. On the other hand: $$ X_1 \leq X_2 + X_3 $$ would ensure that $X_1$ is zero if both are zero.

$\endgroup$
1
  • 1
    $\begingroup$ In your second inequality $\frac{1}{2}$ can be removed. There is also a typo ($X_1$ on the right hand side should be $X_3$) $\endgroup$ Commented Nov 14, 2018 at 13:32

You must log in to answer this question.

Start asking to get answers

Find the answer to your question by asking.

Ask question

Explore related questions

See similar questions with these tags.